Author Topic: EM Drive Developments - related to space flight applications - Thread 2  (Read 3321492 times)

Offline Einstein79

  • Member
  • Posts: 11
  • Liked: 4
  • Likes Given: 0
The reason for the confusion over the violation of classical physics is because this system has nothing to do with classical physics. Moreover, the “thrust” that is being calculated is not thrust at all but space moving the drive from one position to another which can merely be related to thrust but is not, per se, thrust. The controlling factor here is, of course, the resonant frequency. If you match the resonant frequency that space uses to “hold” the object you will develop a “cavity” that the “object will move towards”. The reason why the device cannot be “pushed off of” for conservation of momentum to hold true is because space is already pushing on it satisfying the law.

A couple of postulates to keep in mind that will help with these experiments are:
1. Space creates light.
2. Space itself is a resonating chamber.
Interesting! Would you then be prepared to write down the equations of motion so that we can play with them?

The Mexican hat potential is a good place to start, I think, but not sure yet. It seems reasonable because it might coalesce with symmetry breaking.

You and Mulletron seem to be going in the same direction, you might want to bounce ideas off each other...

YES please! If there are any others trying to tackle this problem this way we certainly need to put our heads together. I think we are being overly literal when speaking of the shape of the resonating cavity. The article that Dr. Rodal quoted stated that Dr. White was creating a warp bubble is on point because this "warp bubble" should be the resonating cavity itself in order to be in agreement with general relativity. Therefore, it does not matter what shape the EM drive is so long as it creates the fields necessary to generate the warp bubble/resonating chamber we need.

Offline Einstein79

  • Member
  • Posts: 11
  • Liked: 4
  • Likes Given: 0
I'm wondering, in the context of dr. rodal's remark on Shawyer's theory, where he (Shawyer) poses that forces are to be observed on the ends only, what would happen if one or both ends were to be replace by Cullen type ring reflector or mesh?
Would one still observe forces of the same magnitude?

From what I understood, waves would still be bouncing yet the surface upon which a force can be applied, or a momentum be transferred to would be drastically reduced...

Somewhere , somehow, if this device really works, there must be a momentum transfer onto the frustum....

If it is uncertain what role the end plates play, why try dielectric materials, or materials with a high magnetic permeability (this subject got completely lost in the current discussion?) on the side walls of the frustum to see if any drag effect is in effect?

This device does really work and there need not be a momentum transfer onto the frustum because the momentum transfer is between the warp bubble and space-time itself. Remember, Dr. White's intention is to create a warp bubble where everything inside this warp bubble (the drive) are unperturbed.

Offline seeingstars

  • Member
  • Posts: 1
  • Liked: 1
  • Likes Given: 1
There was discussion early in the thread of the claimed (once-claimed?  still-claimed?  not sure) mechanism of ExB drift on e+/e- pairs driving the thrust.  This was later muddied a bit by some of March's comments that thrust is not related to the Poynting vector direction after all, but I am still trying to wrap my head around the idea of whether a net ExB drift in a cavity is ever a possibility.  I don't immediately see how you could get a nonzero ExB drift integrated over an RF cycle. 

Statements I think are true:
1) E and B fields in a waveguide or cavity will be mutually perpendicular at all points
2) They will oscillate like sines in time (with more complicated spatial structure, for sure)
3) The ExB drift velocity magnitude will likewise oscillate in time
4) The net thrust produced by causing e+/e- pairs to ExB drift will be proportional to the (double) integral of the cross-product ExB over an RF cycle and over the cavity volume
5) This integral will go identically to zero because you can separate the time and space integrals, and the time integral will go to zero

Mathematically my Jackson-fu is rusty but I think you avoid sin^2 integrals (which would be nonzero) because E and B are perpendicular or out of phase, so in any vector direction you're really integrating a sin(t)*cos(t) term which is equivalent to sin(2t).

The above is a micro view.  From a macro view, the ExB drift is in the same direction as the Poynting vector or radiation pressure, so again I'd expect in classical E&M with conservation of momentum that this would integrate to zero over the interior walls of the cavity.  The only niggle I have there is you're pumping energy into the cavity from a source port, so that energy must flow somewhere to dissipate; perhaps this gives a net energy flow and thus net ExB drift direction.  I don't see where dielectrics come into the picture at all, I'm afraid. 

The point of all this is that I could accept a funny structure in classical E&M that produces net ExB if I could wrap my head around how that's possible, and then say "maybe" to pushing on funny e+/e- pairs pending clear experimental evidence.  But I can't see how you make the cavity to do the pushing in the first place. 
« Last Edit: 05/11/2015 05:18 pm by Chris Bergin »

Offline Mulletron

  • Full Member
  • ****
  • Posts: 1150
  • Liked: 837
  • Likes Given: 1071
We really need to take a stand and not entertain further discussion of "pushing on virtual particles" or referring to the QV as a plasma. The overwhelming scientific consensus is that the quantum vacuum doesn't work this way.
And I can feel the change in the wind right now - Rod Stewart

Offline Rodal

  • Senior Member
  • *****
  • Posts: 5911
  • USA
  • Liked: 6124
  • Likes Given: 5564
We really need to take a stand and not entertain further discussion of "pushing on virtual particles" or referring to the QV as a plasma. The overwhelming scientific consensus is that the quantum vacuum doesn't work this way.

There is also overwhelming consensus that:

1) Conservation of Momentum is a universal principle
2) Conservation of Energy is a universal principle
3) Maxwell's stress tensor components on the lateral surfaces of a conical cavity are not zero (resonating with electromagnetic waves). (There are forces on the conical walls of the EM Drive that cannot be ignored).
4) The principle of operation of the ring laser gyroscope does not justify EM Drive thrust,
5) Electromagnetic waves are not travelling at a speed of only 0.1 c at the small end of the EM Drive
6) Finite Element Analysis based on variational, virtual energy principles (like COMSOL, ANSYS, ABAQUS, etc.) should not predict a net Lorentz force or a Poynting vector for the EM Drive over a whole period or multiples thereof, because FEA satisfies conservation of momentum.
7) etc. etc.

I adhere and support all six of the above.


Why should we not entertain further discussion of any of these topics?

Having to support and defend the above-mentioned arguments and many more, based on technical arguments, experiments and math, (instead of name calling) keeps our brains young and creative.

Hopefully such discussions will lead us to find the truth, and if we don't get there, at least they are fun to discuss  :)
« Last Edit: 05/11/2015 06:08 pm by Rodal »

Offline Mulletron

  • Full Member
  • ****
  • Posts: 1150
  • Liked: 837
  • Likes Given: 1071
Good. We are in violent agreement to self regulate this thread and keep pseudoscience out. Otherwise, this effort is lost.
« Last Edit: 05/11/2015 06:14 pm by Mulletron »
And I can feel the change in the wind right now - Rod Stewart

Offline tchernik

  • Full Member
  • **
  • Posts: 274
  • Liked: 315
  • Likes Given: 641
I wholeheartedly agree with the intention of the smart people around here, helping the rest of us to filter the science from the chaff.

Nevertheless, I assume there is going to be a break point: either the Emdrive proves to deliver some anomalous thrust or it doesn't.

If it does, such anomaly would maybe go against some of the scientific principles stated here (as very pointedly remarked by frobnicat et al).

But such proof by counterexample would need to be really overwhelming and repeatable in order to be accepted. Is that correct?

Offline RonM

  • Senior Member
  • *****
  • Posts: 3340
  • Atlanta, Georgia USA
  • Liked: 2231
  • Likes Given: 1584
Good. We are in violent agreement to self regulate this thread and keep pseudoscience out. Otherwise, this effort is lost.

Perhaps it is time to start a new thread with stricter rules clearly listed in the OP. Point and move other comments to the Feature Article thread. I think that would be fair to everyone.

Anyway, this thread is getting pretty long.

Everyone, keep up the good work. This has been a fascinating and educational thread.

Offline WarpTech

  • Full Member
  • ****
  • Posts: 1407
  • Do it!
  • Statesville, NC
  • Liked: 1453
  • Likes Given: 1925
...
I know a photon is not a ball but my question is, in "Newtonian layman's terms" how does the line of thinking you are developing making that analogy not valid, i.e. imply apparent deviation from conservation of momentum ?

The ball (photon) doesn't fall back down the well. There is nothing to give it back enough energy to do so. It dissipates in multiple reflections between the walls and the big end. They are not getting more out than they put in, so it does not violate conservation of energy. They are simply getting more NET momentum on one direction than in the other direction because there is more dissipation and attenuation in one direction than there is in the other. Dissipative systems are typically "not" conservative, loses prevent a true equal measure from occuring in both directions.

Todd D.

Classically, a dissipative system is conservative for both momentum and energy, it's just that for energy there is a (irreversible) conversion to a degraded form of energy, but there is no such thing as a mysterious part of total energy that would simply vanish. Even if not always convenient, an open system can be seen as part of a bigger closed system, and short of that the deltas total energy and total momentum of an open system can still be accounted, at least in principle, as integrated fluxes exchanged between open system and an outside.

My Newtonian ball of momentum pb can encounter an arbitrarily varying Force Fcb(t) of container on ball (vectors in bold). And dpb/dt=Fcb=-Fbc=-dpc/dt. That is instant conservation of momentum, and obviously integrating on successive instants just yields delta_pb=-delta_pc or  delta_pb+delta_pc=0, conservation of momentum on any time interval whatever the shape of varying Force Fcb(t). Where and how quantitatively your system is showing an apparent breaking of CoM at an "instantaneous scale" dt ? Short of that, details of trajectory is just, ahem, arm waving for propulsion purpose (aka Dean drive).

At the "instantaneous scale"  there are collisions between photons and atoms where momentum is transferred and it generates heat but not thrust. When the photons are injected their momentum is p1 and energy is E1. It can only conserve NET momentum if there is a 50/50 probability that momentum is absorbed in each direction, without generating ANY heat at all. As soon as things start getting hot, the probability is not 50/50 anymore, then some of the momentum is not being absorbed as thrust, but rather to heat up the metal. Therefore, the NET momentum in either direction will depend on the difference in the dissipation and attenuation, in each direction.

Todd D.

Offline Rodal

  • Senior Member
  • *****
  • Posts: 5911
  • USA
  • Liked: 6124
  • Likes Given: 5564
...

At the "instantaneous scale"  there are collisions between photons and atoms where momentum is transferred and it generates heat but not thrust. When the photons are injected their momentum is p1 and energy is E1. It can only conserve NET momentum if there is a 50/50 probability that momentum is absorbed in each direction, without generating ANY heat at all. As soon as things start getting hot, the probability is not 50/50 anymore, then some of the momentum is not being absorbed as thrust, but rather to heat up the metal. Therefore, the NET momentum in either direction will depend on the difference in the dissipation and attenuation, in each direction.

Todd D.
Yes, the problem is, however, how would this, quantitatively, result in a more efficient (thrust/PowerInput) propellant-less drive than a perfectly-collimated photon rocket.

Another fascinating observation that @frobnicat made early on, is that for photons (whether tunneling, dissipation or another mechanism) to end up producing a more efficient drive than a photon rocket, the photons would have to escape the EM Drive as tachyons, superluminally.
« Last Edit: 05/11/2015 07:11 pm by Rodal »

Offline SeeShells

  • Senior Member
  • *****
  • Posts: 2442
  • Every action there's a reaction we try to grasp.
  • United States
  • Liked: 3186
  • Likes Given: 2708
I know I'm new here but I've been in engineering for almost 50 years. The EM drive seems to parallel so many things I've seen in electronics and embrace harmonics and it got me thinking how it would compare to things like a YAGI antenna for gain buy linking them in series. Would you get a Q gain in thrust?

Great thought!
I'm digging on this idea right now but more than Q it seems like the phasing has some profound effects as well. Not a lot of data there, but what little I've gleaned it looks like something I want to pursue. I've been refreshing my old vacuum tube/crystal radio school education so I can wrap my brain around it. I have some ideas but they are not ready to throw out here quite yet.
I found a little blurb in a presentation where Dr. White (Sonny) briefly mentioned one of the tests they were thinking of was to put another passive EM device behind the active one to see if they could discern a pattern. To me this sounded like an YAGI and a great idea. Stack them up like a YAGI I thought, phase them to increase thrust.

Offline Einstein79

  • Member
  • Posts: 11
  • Liked: 4
  • Likes Given: 0
Good. We are in violent agreement to self regulate this thread and keep pseudoscience out. Otherwise, this effort is lost.

This endeavour is by definition "pseudoscience". I.e. it defies all our logic and principles that we have previously and whole heartedly embraced. Space is an actual object and performs work on everything we observe. The "vacuum" does not exist and this experiment is proof of this statement. We must question everything that we have learned and realize that the truth may lie in "pseudoscience". This experiment reveals that we lack understanding of our physical reality and the "laws" we blindly accept as truth are evidence of this misunderstanding because if they were completely accurrate then we would have already solved the problem.     

Offline Rodal

  • Senior Member
  • *****
  • Posts: 5911
  • USA
  • Liked: 6124
  • Likes Given: 5564
I know I'm new here but I've been in engineering for almost 50 years. The EM drive seems to parallel so many things I've seen in electronics and embrace harmonics and it got me thinking how it would compare to things like a YAGI antenna for gain buy linking them in series. Would you get a Q gain in thrust?

Great thought!
I'm digging on this idea right now but more than Q it seems like the phasing has some profound effects as well. Not a lot of data there, but what little I've gleaned it looks like something I want to pursue. I've been refreshing my old vacuum tube/crystal radio school education so I can wrap my brain around it. I have some ideas but they are not ready to throw out here quite yet.
I found a little blurb in a presentation where Dr. White (Sonny) briefly mentioned one of the tests they were thinking of was to put another passive EM device behind the active one to see if they could discern a pattern. To me this sounded like an YAGI and a great idea. Stack them up like a YAGI I thought, phase them to increase thrust.
1) Interesting how that presentation was just a few months ago. That was presented by the Chief Scientist NASA Ames.  How things have changed, given the present official reaction by NASA and NASA Glenn regarding the efforts at NASA Eagleworks.  (The "applications" section of the NSF article that generated so much controversy are old papers by Dr. White that he had published months and years ago).

2) Unfortunately the tiny budget at NASA Eagleworks has prevented them from conducting the proposal to put another passive EM device behind the active one.  Star-Drive posted some time ago that they didn't have another EM Drive to conduct the test  :( 

Offline WarpTech

  • Full Member
  • ****
  • Posts: 1407
  • Do it!
  • Statesville, NC
  • Liked: 1453
  • Likes Given: 1925
...

At the "instantaneous scale"  there are collisions between photons and atoms where momentum is transferred and it generates heat but not thrust. When the photons are injected their momentum is p1 and energy is E1. It can only conserve NET momentum if there is a 50/50 probability that momentum is absorbed in each direction, without generating ANY heat at all. As soon as things start getting hot, the probability is not 50/50 anymore, then some of the momentum is not being absorbed as thrust, but rather to heat up the metal. Therefore, the NET momentum in either direction will depend on the difference in the dissipation and attenuation, in each direction.

Todd D.
Yes, the problem is, however, how would this, quantitatively, result in a more efficient (thrust/PowerInput) propellant-less drive than a perfectly-collimated photon rocket.

Another fascinating observation that @frobnicat made early on, is that for photons (whether tunneling, dissipation or another mechanism) to end up producing a more efficient drive than a photon rocket, the photons would have to escape the EM Drive as tachyons, superluminally.

Where is the calculation for a photon rocket comparison? When calculating the photon rocket thrust was the input power used, or the input power multiplied by the Q? I don't think I've seen that yet. I seriously need to start keeping a spreadsheet of data, tests, results and discussions. It's too much to keep track of in my head.

Offline SeeShells

  • Senior Member
  • *****
  • Posts: 2442
  • Every action there's a reaction we try to grasp.
  • United States
  • Liked: 3186
  • Likes Given: 2708

1) Interesting how that presentation was just a few months ago. That was presented by the Chief Scientist NASA Ames.  How things have changed, given the present official reaction by NASA and NASA Glenn regarding the efforts at NASA Eagleworks.  (The "applications" section of the NSF article that generated so much controversy are old papers by Dr. White that he had published months and years ago).

2) Unfortunately the tiny budget at NASA Eagleworks has prevented them from conducting the proposal to put another passive EM device behind the active one.  Star-Drive posted some time ago that they didn't have another EM Drive to conduct the test  :(
My intuition gave me patents (have more than a few) but the science gave me understanding. My intuition says that you'll find a solution, funding can happen and greater public exposure will help. On this point I will write my congressman and anyone else who can help push, cajole or twist an arm and I would urge anyone else to do. I have no doubt something is there and that something needs good science to resolve it. The investment is not big, but the ROI is the stars (well, maybe the local neighborhood). ;)
« Last Edit: 05/12/2015 04:31 am by Galactic Penguin SST »

Offline Rodal

  • Senior Member
  • *****
  • Posts: 5911
  • USA
  • Liked: 6124
  • Likes Given: 5564
...
Where is the calculation for a photon rocket comparison? When calculating the photon rocket thrust was the input power used, or the input power multiplied by the Q? I don't think I've seen that yet. I seriously need to start keeping a spreadsheet of data, tests, results and discussions. It's too much to keep track of in my head.

See:

1) http://forum.nasaspaceflight.com/index.php?topic=36313.msg1302455#msg1302455

2) http://forum.nasaspaceflight.com/index.php?topic=29276.msg1276053#msg1276053

« Last Edit: 05/11/2015 07:36 pm by Rodal »

Offline zlspradlin

  • Member
  • Posts: 6
  • USA
  • Liked: 3
  • Likes Given: 2
I'm new here and I just thought I would post this video for you all.  Its a very VERY sloppy experimental setup of something like the what people on this forum are talking about.  The interesting thing here is the man in the video doesn't use end-plates and its quite a bit slimmer than the EM Drive.  Here is the video:

youtube.com/watch?v=vcaOKX7Ko7w

What are some thoughts about the video posted?

Rough translation (From original in Russian):

Quote
Created Shawyer  (EM Drive) engine is very easy and simple in its design . It provides the necessary thrust " by the oscillation of the microwaves inside the vacuum container ."
http: //hi-news.ru/technology/v-nasa-i ...
I decided that the system should not be closed



It is a waveguide with one end open.  The Russian author points out (later in his Russian text) that he thinks that Shawyer, and others are wrong in using a closed cavity.

The reference (Cullen) given by Shawyer to support his theory also used in his experimental measurements of pressure, a cavity with one end open (with a transparent glass)


It is known that a microwave waveguide having one end open will display directional thrust, as the microwave photons escape the waveguide.  The problem with the EM Drive is that it is a closed cavity, hence it cannot be explained solely based on Maxwell's equations. Something else is needed: General Relativity, QV,  something else.

A waveguide with one end open will behave as a very inefficient photon rocket: thousands of times less thrust per power input than what is claimed in the EM Drive experiments.

Would it be possible to amplify the waves in the cavity, trying to create a high pressure scenario that perhaps could be an efficient "photon" rocket? Can these waves be pressurized? Is this pointless in speculating at this point in time?

I had a thought over the weekend and saw that at least one other poster had posted about CMB (cosmic microwave background) but that discussion quickly died and I'm not sure why. Seeing as CMB is basically homogenous and permeates in all directions couldn't this help explain the thrust? Perhaps "warping" (for lack of a better word) around the CMB, or the CMB around the drive?

Offline RotoSequence

  • Senior Member
  • *****
  • Posts: 2208
  • Liked: 2068
  • Likes Given: 1535
Good. We are in violent agreement to self regulate this thread and keep pseudoscience out. Otherwise, this effort is lost.

This endeavour is by definition "pseudoscience". I.e. it defies all our logic and principles that we have previously and whole heartedly embraced. Space is an actual object and performs work on everything we observe. The "vacuum" does not exist and this experiment is proof of this statement. We must question everything that we have learned and realize that the truth may lie in "pseudoscience". This experiment reveals that we lack understanding of our physical reality and the "laws" we blindly accept as truth are evidence of this misunderstanding because if they were completely accurrate then we would have already solved the problem.     

This is a strange definition of pseudoscience, and seems somewhat distant from its actual meaning. Pseudoscience involves false, or otherwise inaccurate claims of adherence to the scientific method.
« Last Edit: 05/11/2015 07:59 pm by RotoSequence »

Offline Flyby

  • Full Member
  • ***
  • Posts: 388
  • Belgium
  • Liked: 451
  • Likes Given: 48
I'm wondering, in the context of dr. rodal's remark on Shawyer's theory, where he (Shawyer) poses that forces are to be observed on the ends only, what would happen if one or both ends were to be replace by Cullen type ring reflector or mesh?
Would one still observe forces of the same magnitude?

From what I understood, waves would still be bouncing yet the surface upon which a force can be applied, or a momentum be transferred to would be drastically reduced...

Somewhere , somehow, if this device really works, there must be a momentum transfer onto the frustum....

If it is uncertain what role the end plates play, why try dielectric materials, or materials with a high magnetic permeability (this subject got completely lost in the current discussion?) on the side walls of the frustum to see if any drag effect is in effect?

This device does really work and there need not be a momentum transfer onto the frustum because the momentum transfer is between the warp bubble and space-time itself. Remember, Dr. White's intention is to create a warp bubble where everything inside this warp bubble (the drive) are unperturbed.

may i bring in a few small but significant corrections?

The device does not" really work", but it SEEMS to work...
Perception is a different thing then certitude..

There are precedences of test setups that appear to be working, yet have not been able to defend themselves very well again other possible solutions. The proof of its working is still fragmentary and most certainly needs further testing to validate it is actually functioning.
I'm not a "believer" or "non-believer" but it is about gathering enough irrefutable evidence to give credibility to the device.
I do not want to see a smoking gun, i want to hear and see the gunshot... (hence my anticipation for the upcoming July test at EW)

As for the warp bubble, it is just - and nothing more - one of the possible mathematical constructs/theories that try to give a theoretical framework for the observations.
And theories can be debates endlessly as long there is no hard experimental data.
In the end it is experimental data that defines the preferred theory that gives a satisfactory explanation...



On a personal level, being thought only old school newtonian physics, I do have some reluctance to accept quantum mechanics, but i wont exclude it as a possibility...

As it is now, my preference goes to Todd's theoretical model, simply because that's something i can still understand and it makes sense to me, as layman. Not sure if it is THE right theory, but at this stage we're in , that doesn't matter much...(yet)

Offline Rodal

  • Senior Member
  • *****
  • Posts: 5911
  • USA
  • Liked: 6124
  • Likes Given: 5564
...
Would it be possible to amplify the waves in the cavity, trying to create a high pressure scenario that perhaps could be an efficient "photon" rocket? Can these waves be pressurized?
The waves are electromagnetic waves with photons at a microwave frequency.  They can be Maser [ https://en.wikipedia.org/wiki/Maser ] which is very expensive -beyond the reach of the NASA researchers' budget- and whose goal would be unclear, given the present understanding.



...Totally random thought but what if y = e
e is not random  :)  (e is like an encrypted message: its digits are in a random sequence, but it contains a huge amount of information, if one has the key  :) )

This is not random either, it is beautiful: 



...As it is now, my preference goes to Todd's theoretical model, simply because that's something i can still understand and it makes sense to me, as layman. Not sure if it is THE right theory, but at this stage we're in , that doesn't matter much...(yet)
We are very lucky to have Todd and all other newcomers join our forum.  It is delightful to see how clearly Todd writes about GR, and the flood of new ideas and experience being shared in this thread.  All the pain and suffering resulting from the commingling of the NSF article with the fringe articles  was well worth it if as a result of it we received a flood of new blood with fresh eyes and fresh brains on this matter  :)
« Last Edit: 05/11/2015 08:44 pm by Rodal »

Tags:
 

Advertisement NovaTech
Advertisement Northrop Grumman
Advertisement
Advertisement Margaritaville Beach Resort South Padre Island
Advertisement Brady Kenniston
Advertisement NextSpaceflight
Advertisement Nathan Barker Photography
0